Want to compare multiple-choice answers? (1 Viewer)

XcarvengerX

Chocobo
Joined
Oct 31, 2005
Messages
378
Location
Sydney
Gender
Male
HSC
2006
@matty fwd - Question 9 is the only one upped for debate.

I don't know about homopolar or whatever generator but I think we only need to know AC and DC Generators for the HSC Physics syllabus, unless I missed a dotpoint or two in section 9.1 of syllabus or third column.
 

yankyfly

New Member
Joined
Oct 18, 2005
Messages
26
Gender
Male
HSC
2006
suchet_i said:
i got 15/15 if i got Q9 rite .. i put down C ... and reading back again and again and even after looking at explanations here i still think its C

9 is not C. Using the right hand rule the magnetic field goes from north to south. And the motion through it from right to left. So thr conventional current gets pushed down and around. The only contention could be as to whether no current was flowing at all. But since everyone else here has confirmed that current flows, the answer is B. even my physics teacher said it was b
 

XcarvengerX

Chocobo
Joined
Oct 31, 2005
Messages
378
Location
Sydney
Gender
Male
HSC
2006
Good then. ;)
This is the most correct answer according to the majority of BoSers. Take this as official until the Board of Studies releases the actual answers. So if you want to mark yours, use this:
1. C
2. B
3. D
4. B
5. D
6. A
7. D
8. A
9. B
10. A
11. A
12. C
13. D
14. C
15. A
 

StYlez-

New Member
Joined
Aug 26, 2005
Messages
20
Gender
Male
HSC
2006
someone explain to me why 10 is A again, i still dont get it.
 

Desiree

New Member
Joined
Mar 11, 2006
Messages
11
Gender
Female
HSC
2006
  1. C
  2. C
  3. D
  4. B
  5. C (i couldn't decide between C and D, enny menny minny moe)
  6. A
  7. B
  8. B
  9. C
  10. B
  11. A
  12. C
  13. D
  14. C
  15. B (enny menny)
 

Lachdog

New Member
Joined
Oct 28, 2006
Messages
7
Gender
Male
HSC
2006
Yeh i don't understand how 10 could be a either?

Transformers run with AC.

My answers were: 1 c 2 b 3 d 4 b 5 d 6 a 7 d 8 a 9 d 10 d 11 a 12 c 13 d 14 c 15 a.
 

Lockhart

Fugitive/assasin
Joined
Nov 30, 2005
Messages
44
Location
Sydney
Gender
Male
HSC
2006
Altough I put 10 d as well, its wrong. 10 is a as while the transformer is turned on there is back emf in the wire which means the voltage will gradually rise to a maximun. This constitutes a changing magnet flux which will cause voltage in the secondary coil when it is being turned on and off.
The cell, is a battery therfore it must be DC. Which means while its running it will have a constant voltage.
 

Lockhart

Fugitive/assasin
Joined
Nov 30, 2005
Messages
44
Location
Sydney
Gender
Male
HSC
2006
Again while I put B for question 8 it is actually A.
magnetic flux is defined by (B{perpendicular to}A) therefore when the coil is initially at right angles magnetic flux is a maximun. When it is parrelle, no area is being threaded through the field so A becomes zero and flux is zero at 90 degrees.
 

Lockhart

Fugitive/assasin
Joined
Nov 30, 2005
Messages
44
Location
Sydney
Gender
Male
HSC
2006
the only question that is in doubt is question 9.
I personally put A.
This appartatus is the same that functions in a electromagnetic break, it produces eddy currents within the wheel.
What I fail to see is why these currents should travel trough a loaded wire at all, where there is greater resistance. shouldn't they remain within the wheel travelling in two circles completeing the circiut and releasing heat by doing so.
What reason is there to sugest that the current will move through the wire.
That was my reasoning, however some people came up with an interent site suggesting that this does produce a current. In which case I don't doubt that i'm wrong. But if anyone could explain the flaw in my reasoning i would be grateful.
 

Lockhart

Fugitive/assasin
Joined
Nov 30, 2005
Messages
44
Location
Sydney
Gender
Male
HSC
2006
just for anyone whiose interested heres a rough sample of how most of us seem to have gone in the exam.

1. C C C C C C C A C C C
2. B B B B B B B B B B B
3. D D D D
C D D D D D D
4. B B B B
A B B B B B B
5. D D D D
B D D D B D D
6. A A A A A A A A A A A
7. D D D C B D C B B C D
8.
B C B A B A A A B A A
9.
A A B B D D B B B D C
10.
D A D B B D A A B D A
11. A A A A A A A A A A A
12. C C C C C C C C C C C
13. D D D D D D D D D D D
14. C C
A C C C C C D C C
15.
B A A A A A D A D A A

1. C C C C C C C C C B C
2. B B
D B B B B B B A B
3. D D D D D D D D D
C D
4. B B B B B
C B B D C B
5. D D D D D D
C D A B D
6. A
C A A A A A A A B A
7. D
C D D C C C C C D D
8.
B B B A A B A A A B A
9. B
D B D A A B B B A B
10. A
D A A A D A D D C A
11. A A A A A A A A A
B A
12. C C C
D C D C C C A C
13. D C D D D D D D B D
D
14. C C C C C C C C
B A C
15. A
B A A B A A A A C A

Green are correct answers with perhaps question 9 being different

Red are when students got a wrong answer

So there were out of the first 21 students to reply:

5 fourteens
6 thirteens
2 twelves
1 eleven
2 tens
3 nines
1 two
 

shinji

Is in A State Of Trance
Joined
Feb 2, 2005
Messages
2,733
Location
Syd-ney
Gender
Male
HSC
2006
lol, i got either 12-13/15 for mc
13 if 9 is b ^_^
 

XcarvengerX

Chocobo
Joined
Oct 31, 2005
Messages
378
Location
Sydney
Gender
Male
HSC
2006
Nice Lockhart. Thanks for tabulating them. Note that there are some incorrect answers that you didn't make it red. (question 7 and 13).

Questions 7, 8, 9 and 10 pose to be the most problematic ones.
So no one got full mark?
 

meekzy89

New Member
Joined
Mar 28, 2006
Messages
13
Gender
Female
HSC
2006
1. c
2. b
3. d
4. b
5. b [i kno this is wrong]
6. a
7. c
8. b
9. c [my hand rule didnt work.. or something]
10. a [i think its wrong]
11. a
12. c
13. d
14. c
15. a

umm i think i got 11/15... dammit!!
 

fizzwizz

New Member
Joined
Nov 5, 2005
Messages
17
Gender
Undisclosed
HSC
N/A
Hi The correct answer to this q is B I think.Use Lenz's law to answer it, and we will take answer C to show that it is incorrect.

If current goes through globe from X to Y then a current moves upwards from brush Y to the handle. Now think of this current as a current in a single conductor pointing upwards, passing through the magnetic field which is directed from left to right. Now use the right hand push rule: current up, fingers pointing left to right................this produces a force on the conductor (disc) into the page. But it is already moving into the page (at the bootom) so this force would increase its jmotion, not decrease its motion as required by lenzs law. A force has got to act on the conductor pushing it out of the field, as it enters the field. Using current flow downwards from handle to brush Y gives you this force. So answer is B.

:)
 

switchfootfan

New Member
Joined
Sep 8, 2005
Messages
28
Gender
Male
HSC
2006
yeah thinking around 11-13 for mc]
1c
2b
3d *i think i put that*
4b
5a (i know i screwed up- dont know how i got that....)
6a (its either a or b and i dont think it attracts)
7b (i think its c, your right)
8a (its flux)
9borc (cant remember- think it might have been b that i put- hope so)
10c (this is open to debate- i pritty sure you can get dc transformers)
11a (obviously)
12c
13d
14c (E=v/d and perpecx does not conduct electricity)
15a (v=wavelegthx frequency)
thats what i got
 

meekzy89

New Member
Joined
Mar 28, 2006
Messages
13
Gender
Female
HSC
2006
fizzwizz said:
Hi The correct answer to this q is B I think.Use Lenz's law to answer it, and we will take answer C to show that it is incorrect.

If current goes through globe from X to Y then a current moves upwards from brush Y to the handle. Now think of this current as a current in a single conductor pointing upwards, passing through the magnetic field which is directed from left to right. Now use the right hand push rule: current up, fingers pointing left to right................this produces a force on the conductor (disc) into the page. But it is already moving into the page (at the bootom) so this force would increase its jmotion, not decrease its motion as required by lenzs law. A force has got to act on the conductor pushing it out of the field, as it enters the field. Using current flow downwards from handle to brush Y gives you this force. So answer is B.

:)
HAHAHAHAHAHAHA!!! OMG i know what ive done! .. i used my left hand.. cuz i was friken WRITING with my right
LOL!!!!!

omggg can i get any more stupider! BAHAHA!!!
thankuu
 

XcarvengerX

Chocobo
Joined
Oct 31, 2005
Messages
378
Location
Sydney
Gender
Male
HSC
2006
7 is D because you have to use pythagoras to calculate the diagonal length AND also sin 90 NOT sin 45. If you use either pythagoras but sin 45 OR sin 90 but not pythagoras, then you get C which is incorrect.
 

eX-Bhai

Member
Joined
Mar 15, 2005
Messages
78
Gender
Male
HSC
1998
XcarvengerX said:
7 is D because you have to use pythagoras to calculate the diagonal length AND also sin 90 NOT sin 45. If you use either pythagoras but sin 45 OR sin 90 but not pythagoras, then you get C which is incorrect.
that is incorrect. The answer is C

By using basic trig, Sin 45 = 0.4 / L
Therefore 0.4 = L.sin45

By using the formula F = BILsin@

B = 0.5 T
A = 3 A
Lsin@ = 0.4

Therefore F = 0.6 N
 

XcarvengerX

Chocobo
Joined
Oct 31, 2005
Messages
378
Location
Sydney
Gender
Male
HSC
2006
eX-Bhai said:
that is incorrect. The answer is C

By using basic trig, Sin 45 = 0.4 / L
Therefore 0.4 = L.sin45

By using the formula F = BILsin@

B = 0.5 T
A = 3 A
Lsin@ = 0.4

Therefore F = 0.6 N
The bold line is the part where you got it wrong. Look at the diagram again. The conductor is already perpendicular to the magnetic field line, therefore you use sin 90. The 45 degree is to indicate that the conductor is indeed passing through as the diagonal of the square magnetic field.
 

Users Who Are Viewing This Thread (Users: 0, Guests: 1)

Top